Difference between revisions of "2021 WSMO Team Round Problems/Problem 4"
Line 6: | Line 6: | ||
==Solution== | ==Solution== | ||
Note that <math>A_1B_1C_1</math> is a right triangle with right angle at <math>B_1</math>, so its area is | Note that <math>A_1B_1C_1</math> is a right triangle with right angle at <math>B_1</math>, so its area is | ||
− | <cmath>[A_1B_1C_1] = \frac{1}{2}\cdot A_1B_1\cdot B_1C_1 = \frac{9\sqrt{3}}{2}</cmath> | + | <cmath>[A_1B_1C_1] = \frac{1}{2}\cdot A_1B_1\cdot B_1C_1 = \frac{9\sqrt{3}}{2}.</cmath> |
For all <math>i</math>, we have | For all <math>i</math>, we have | ||
− | <cmath>\frac{A_iC_i}{A_{i-1}C_{i-1}} = \frac{B_{i-1}C_{i-1}}{A_{i-1}C_{i-1}} = \frac{B_1C_1}{A_1C_1} = \frac{3\sqrt{3}}{6} = \frac{\sqrt{3}}{2}</cmath> | + | <cmath>\frac{A_iC_i}{A_{i-1}C_{i-1}} = \frac{B_{i-1}C_{i-1}}{A_{i-1}C_{i-1}} = \frac{B_1C_1}{A_1C_1} = \frac{3\sqrt{3}}{6} = \frac{\sqrt{3}}{2}.</cmath> |
Thus, the ratio of areas is | Thus, the ratio of areas is | ||
− | <cmath>\frac{[A_iB_iC_i]}{[A_{i-1}B_{i-1}C_{i-1}]} = \left(\frac{\sqrt{3}}{2}\right)^2 = \frac{3}{4}</cmath> | + | <cmath>\frac{[A_iB_iC_i]}{[A_{i-1}B_{i-1}C_{i-1}]} = \left(\frac{\sqrt{3}}{2}\right)^2 = \frac{3}{4}.</cmath> |
This forms a geometric series: | This forms a geometric series: | ||
− | <cmath>\sum_{i=1}^\infty [A_iB_iC_i] = [A_1B_1C_1] \sum_{i=0}^\infty \left(\frac{3}{4}\right)^i = \frac{9\sqrt{3}}{2} \cdot \frac{1}{1 - \frac{3}{4}} = \frac{9\sqrt{3}}{2} \cdot 4 = 18\sqrt{3}</cmath> | + | <cmath>\sum_{i=1}^\infty [A_iB_iC_i] = [A_1B_1C_1] \sum_{i=0}^\infty \left(\frac{3}{4}\right)^i = \frac{9\sqrt{3}}{2} \cdot \frac{1}{1 - \frac{3}{4}} = \frac{9\sqrt{3}}{2} \cdot 4 = 18\sqrt{3}.</cmath> |
The final answer is | The final answer is | ||
− | <cmath>(18\sqrt{3})^2 = 324\cdot3 = \boxed{972}</cmath> | + | <cmath>(18\sqrt{3})^2 = 324\cdot3 = \boxed{972}.</cmath> |
~pinkpig | ~pinkpig |
Latest revision as of 13:03, 9 September 2025
Problem
Consider a triangle satisfying
. For all successive triangles
, we have
and
, where
is outside of
. Find the value of
where
is the area of
.
Proposed by pinkpig
Solution
Note that is a right triangle with right angle at
, so its area is
For all , we have
Thus, the ratio of areas is
This forms a geometric series:
The final answer is
~pinkpig